User Avatar
jmanasco12783
Joined
Apr 2025
Subscription
Free
User Avatar
jmanasco12783
Saturday, Oct 09 2021

For #10- Strengthening Question, The dentist cites as evidence 5 studies showing that tooth issues in children are proportionally lower in Europe (where water is not fluoridated) than the USA (where water is fluoridated), and argues on the basis of that evidence that fluoridation of water does not substantially prevent tooth decay.

B is tricky, because they bait you to assume that because Americans use fluoride in their toothpaste and still have more tooth problems, the dentist's argument holds. However, his argument was about the fluoridation of water; he never mentions toothpaste. Whether or not Americans use a certain toothpaste doesn't strengthen the argument that fluoridation in water has no substantial impact on tooth problems.

E is correct, because it strengthens the dentist's argument by "blocking" a rebuttal. I almost imagined the dentist making this argument, and someone speaking out to say " Wait, it's not that fluoridation doesn't help, its that Americans have worse teeth because of their diet!" E takes that rebuttal away, and therefore strengthens the argument.

17- Flaw- The author assumes a lot here, but I think the key is ID'ing the conclusion. The author concludes that because the # of students passing last year was well below average, the new curriculum caused instruction quality to decrease. The referential phrase "This" draws our attention back to "well below avg". Okay, but how are we to know that the curriculum impacted instruction? What if this year's class was just really inattentive, or far less dedicated than previous classes? Or, what if the national average is high because everyone else cheats, and this Institute is the only honest institute? How do we know that this Institute's instructors were not always below average? There are too many assumptions made.

B is incorrect because the author never addresses whether or not quality of instruction has increased; they simply argue that because test results were far below average, quality of instruction has decreased. This flaw is inapplicable here because they never cite evidence of the sort mentioned in AC B.

C captures the flaw; the author cannot argue that solely because something is below average, the instruction quality must be diminishing. There could be many other reasons why only 1/3 passed last year, and it could be that it was always below average. Or the nat'l avg increased drastically, and they stayed the same.

19- Again, I think the key to this question is to ID the conclusion. The author concludes that a business that has increased reliance on computers has probably not increased productivity, because productivity dropped in the 60s and 70s as computer tech became widespread/productivity growth has dropped most in industries that heavily rely on computer tech. To weaken this, we would need to attack the support structure of the argument.

A- sort of explains why the phenomenon has occurred, but doesn't weaken the way the conclusion (productivity growth has probably not increased by using computer tech.) is supported by the premises (drop in the 60s and 70s, drop in heavily reliant industries)

D- Undermines the premises- conclusion relationship: It says that growth has been greatest in industry that invested heavily in computer tech. The key for me to this one is to realize that just because productivity has been demonstrated as declining, doesn't mean that any business that increased its reliance on computer tech probably didn't experience growth

Sorry for the long-winded response but hope this helps!

User Avatar

Thursday, Oct 07 2021

jmanasco12783

Test Change-Looking for advice

I'm looking for thoughts on my current situation-- I'm a first-time test taker, and I have studied consistently since around the end of May. I planned on taking the LSAT once, and maybe twice if I drastically underperformed. Sadly, I have let the Nov. date get away from me.

Originally, my plan was to test in October since I wanted to get applications in around Thanksgiving-- but now I'm wondering if I should reschedule. My goal score was a 165, and until about two weeks ago I was PT'ing consistently in the low 160s, with a -4 or -6 on LR, -4/-5 on RC.

Recently though, something clicked for me in LR, and I've gone -0/-1 several times, along with several -1 to -3 scores on RC. My last PT (92) was my best, a 168 with LR -1, RC -3, and LG -7--which is actually one of my best LG sections to date. LG has killed my score since the very beginning and it is so frustrating.

Anyway, I say all that to ask, in light of my recent improvements-- should I pay the fee, postpone to Nov., and try to find some help with LG, or just stick with October? I hate wondering how much higher my score could have been with a better LG performance, but I don't know if I have enough time to improve any on LG/ if the time I'd lose on early application submission is worth it... anyway, any help/advice is appreciated!

Confirm action

Are you sure?